nice cr

This topic has expert replies
Legendary Member
Posts: 833
Joined: Mon Aug 04, 2008 1:56 am
Thanked: 13 times

nice cr

by vivek.kapoor83 » Tue Oct 07, 2008 4:20 am
solve
Attachments
19.jpg

Junior | Next Rank: 30 Posts
Posts: 16
Joined: Tue Jul 29, 2008 12:13 pm
Thanked: 1 times

by akg_gmat » Tue Oct 07, 2008 5:20 am
IMO: C
Last edited by akg_gmat on Tue Oct 07, 2008 7:11 am, edited 1 time in total.

Master | Next Rank: 500 Posts
Posts: 280
Joined: Tue Sep 30, 2008 4:18 am
Thanked: 5 times
GMAT Score:610

by Jatinder » Tue Oct 07, 2008 6:12 am
IMO C
Negate this and the argument falls appart

User avatar
Master | Next Rank: 500 Posts
Posts: 157
Joined: Tue Oct 07, 2008 5:47 am
Thanked: 3 times

by PussInBoots » Tue Oct 07, 2008 7:40 am
Jatinder wrote:IMO C
Negate this and the argument falls appart
I like your explanation, short&simple.

Legendary Member
Posts: 1159
Joined: Wed Apr 16, 2008 10:35 pm
Thanked: 56 times

by raunekk » Tue Oct 07, 2008 9:23 am
imo:c

as said above: negation rule works best..

Legendary Member
Posts: 833
Joined: Mon Aug 04, 2008 1:56 am
Thanked: 13 times

by vivek.kapoor83 » Tue Oct 07, 2008 10:00 am
whts wrong with B ?

Legendary Member
Posts: 833
Joined: Mon Aug 04, 2008 1:56 am
Thanked: 13 times

by vivek.kapoor83 » Tue Oct 07, 2008 10:07 am
Negating B also works. i think

Master | Next Rank: 500 Posts
Posts: 377
Joined: Wed Sep 03, 2008 9:30 am
Thanked: 15 times
Followed by:2 members

by schumi_gmat » Tue Oct 07, 2008 10:41 am
Negating B. supports the conclusion.

Negation -
If Percentage who attend college is not higher now......then by increase in the number of people to attend will increase the average salary. Hence it supports the arg.

ANS C

Legendary Member
Posts: 833
Joined: Mon Aug 04, 2008 1:56 am
Thanked: 13 times

by vivek.kapoor83 » Tue Oct 07, 2008 12:41 pm
cn u also explain howcm c????????with the explantion ..how u negate that

Legendary Member
Posts: 1159
Joined: Wed Apr 16, 2008 10:35 pm
Thanked: 56 times

by raunekk » Tue Oct 07, 2008 11:49 pm
using d negation rule on C


c:the higher average salary for jobs requiring a college degree is due largely to a scarcity among the kravonias workforce of people with a college degree

thus it says that the higher average salary is due to the scarcity of college degree people and not because the number of college dergree people are more...

Thus if this happens..
i.e if the number of college degree people increases=>the average salary will decrease..
this goes against the conclusion...thus the argument falls apart.

thx

Legendary Member
Posts: 1169
Joined: Sun Jul 06, 2008 2:34 am
Thanked: 25 times
Followed by:1 members

by aj5105 » Thu Oct 09, 2008 9:37 am
I chose D.

I negated D and thot it broke the argument.

Master | Next Rank: 500 Posts
Posts: 344
Joined: Sun Sep 28, 2008 11:00 am
Location: USA
Thanked: 6 times
Followed by:1 members

by Bidisha800 » Thu Oct 09, 2008 8:17 pm
Clearly (C)

Master | Next Rank: 500 Posts
Posts: 300
Joined: Sat Aug 22, 2015 10:33 am

by kris77 » Thu May 12, 2016 12:54 pm
Answer C seems to be logical one out of other answer choices